You are on page 1of 61
L.A conducting red of length / is pivoted at one end and rotates with frequency f revolutions per second. A uniform magnetic field B is perpendicular to the plane of rotations. Find the potential ditlerence between the two ends of the red. 1. Two concentric conducting spheres are of radii r) and rg (mr) < rg). They are at a potential difference of V. The space between the spheres is half-filled by a hemispherical shell of dielectric (of diclectric constant €) as shown in the figure below. (a) Determine the electric field E everywhere between the two spheres. (b) Determine the electric displacement D everywhere between the two spheres. ‘SS Q 1. A dipole P is oriented normal to and a distance d from an infinite conducting plane. The plane is at ground (@ = 0). Calculate the magnitude and direction of the foree exerted by the plane on the dipole. L(a} Write down the complete Maxwell's equations in either the Gaus- sian or the MKS system of units. (b) If magnetic monopoles were discovered, Maxwell's equations would be changed. Give these new equations. (c) Use the new equations to derive the equations which represent conservation of electric and magnetic charges which we know classically can neither be created or destroyed. (d) Derive the equation of continuity of energy density of an electro- magnetic field in a region of space where these are no charges. L. Consider a Legrangian density [ (¢ oe a): * Org? Ot 5 / Ldaedt =0 a aL a Ob aL aa (wy * Bacay aa oxy, (a) Show that for any point (x, #). (b) Por a time independent electrostatic potential (x) the Lagrangian density is E-E Sa Show that the uaul Maxwell equation relating EB and p is deriv- L= — po. able from the above extremum principle. 1. Several useful expansions for are a Arr’ ig yee an = Donne (9.0) ¥i5, (8.9) ¢ ) Consider an arbitrary but finite charge distribution, p(t) and let the origin be conveniently located within the distribution. Pp (a) Write down a simple integral expression (not a series) for the electric potential ¢(r) at a point P. (b) Give an expansion for d(r) and identify the monopole, dipole, and quadrpole moments of the charge distribution. (c) Prove that the dipole moment p, of a charge distribution is in- dependent of the origin whenever the net charge is zero. (d) The potential, electric field, and the force due to an external field on a point dipole at the origin are per oa= a 3r| Tr) 5, - 22 ) P r F=p: VE Derive two of these expressions. Folulars which you may find helpful are: ae 7 sind V(A- B)=(A: V)B+(B- VJA+Ax (Vx B)+ Bx (Vx A). L. Two thin lenses of focal lengths fy) and fo are separated by a dis- tance D which is small compared to either focal length. The prin- ciple axes of the two lenses are displaced by a distance d, as drawn: \ | | I . t ‘| (|? 1 D Find the position of the focal point of the combination. For con- ventlence, use an x-y cartesian coordinate centered on lens 1 to express your answer. L. An electric dipole p is at distance d from a grounded conducting (infinite) plane. Use the method of image charges to compute the work required to rotate the dipole from a normal to a parallel orientation relative to the plane. Hints: The electric field due to a dipole h is 3(r - hjr—r-h pole broek Pe and the torque experienced by a dipole h in an external electric field E is h x E. L. A model for an electron consists of a shell of charge distributed uni- formly on the surface of a sphere of radius A. The electron moves with » < c. Find the value of A such that the total momentum earried by the field is just equal to the mechanical momentum mv of the electron. Hints: Be sure to expand in powers of v/e and drop terms whenever you can relative to more important terms at every stage in the solution. L. Note: The following notation is used where the 3 components are displayed as in r = (x,y, 2) where ¥ is the position vector. (a) A vector potential is given by : B A(r) = a a/3 where Bp is a constant. Find the associated magnetic field Bir). and find a gauge transformation that transforms A(r) to the Coulomb or transverse gauge. —y,.n—2,y—2) (b) A vector potential is given by’ A(t) = Cla? — yt at typ 2 ty $27) where C is a constant. Find the associated magnetic field and find a gauge transformation to the Coulomb or transverse gauge. (c) Does the magnetic tield found in the previous part represent a physical magnetic Held, and why or why not? l(a) For a particle with mass m and velocity u, the relativistic energy and momentum are respectively: Demonstrate that only one produces the correct energy and mo- mentum. (b) Add an interaction term, —g(V—u - A), to the appropriate free particle Lagrangian and show that this repreduces the Lorentz force law. L. Recall the definition of the electromagnetic field tensor, pu _ (28° 28") Ort Ax” where A" = (o/c, A) and 2" = (cf, x) are four-vectors, and pa, v is 0,1,2, or 3. (a) Write out the coimponents of F!” in terms of the components (B,, Ey, EB.) and (.B,, B,, B,) of the electric and magnetic fields. (b) Write the Lorentz transformation matrix Lf from one inertial reference frame © to another inertial reference frame 5’, where His moving with velocity v in the x-direction with respect to B (c) Use your result from the previous part to transform F*’ in © to (F" in ©’. Write down the formulas for EB’ and B’ in ©’ in terms of E and B in 3. (d) Now find the force on a moving charge g with velocity v in ther presence of E and B fields. Proceed as follows. First, find the force on the particle in its rest frame, assuming that only an electric field E’ exists in that frame. Then, transform back to the reference frame in which the particle is moving with velocity v. and show that the force in that frame is given by the Lorentz force law. L. Please answer the following: (a) Find the capacitance of a capacitor consisting of two concentric spherical shells with inner radius r and outer radius A. (b) What is the capacitance if a medium with dielectric constant ¢ js placed between the two spherical shells of the previous part. (c) What is the capacitance of the capcitor of part (a) if r is finite and RB = oo. (d) What is the capacitance of the capacitor in part (a) if Ror < r? Is your result reasonable? L. The sealar and vector potentials of an oscillating dipole with dipole moment pp and frequency w are respectively: Vir, 6,4) = — (=) (~) sin lw (* — ")| A(r.8.t) = (AZ) sin [a (t- 5) ] (a) Show that these potentials are in the Lorentz gauge. (b (c ( and ) ) Find the electric and magnetic fields and the Poynting vector. ) Find the total power radiated. Ms d) Speenlate why the sky is blue. L. A plane transverse electromagnetic wave travels through a uniform plasma consisting of cold nonrelativistic electrons and an equal density, m, of immobile singly-charged positive ions. (a) Neglecting any collisional effects, derive the phase velocity of the wave as a function of frequency. (b) Consider a plane boundary separating a region of constant plasma density from a vacuum region. A plane transverse electromag- netic wave with w > \/ne#/epm, is incident on the boundary from the vacuum side making an angle a with respect to the normal to the boundary surface. For what values of a will the wave be completely retlected'? L. Two identical air capacitors are connected in series to a battery of voltage, V. The wire between them, of course, arrives at some potential. Then a dielectric constant, €, is inserted between teh plates of one of the capacitors. What is the change in the potential of the wire? L. An infinitely long uncharged conducting pipe of radius A whose axis coincides with the z-axis is placed into a uniform electric field, E = Ex, Ep = constant. (a) Given that the induced surface charge density on the pipe is g = aycos@, where op is a constant, find the vector force per unit length on the « > 0, y = 0 (ie. O< @ < w/2) part of the pipe. See the figure below. (b) Find the scalar potential outside the pipe. In this case a general solution of Laplace’s equation is @ = ay + bolnp+ S- anp" sin(nd + ay) + s bap “sin({ne + Gn) a-l n=l (c) Find the surface charge density on the pipe, evaluating ¢o in the process. uv lav au (note: VU = Pop + ooo + a5 ) 1. Attached to the starting motor of your car is a solenoid switch. When you turn your ignition key, current J from the battery runs through the solenoid with length and n turns per unit length, creating a magnetic field which acts on a steel plunger (length ¢, permeability yz), thereby engaging the motor. Calculate the force on the plunger as a function of x (neglect fringe fields). . Model a charged particle as a hollow shell of radius a with charge & distributed on its surface. If the particle is moving slowly, then to lowest order in the velocity, v, the electric field is that of a particle at rest, and the magnetic field is B = tv x E. (a) Compute the total energy of the electromagnetic field of the particle. (b) Compute the total momentum of the electromagnetic tield. In both computations, keep only the terms of lowest order in wv. L. All of the following refer to E& M in vacuum. (a) In terms of the 4-vector potential, what is the field strength tensor Fy,” (b) In terms of the electric and magnetic fields, what are the elements of F,.? (c) In terms of the electric and magnetic fields. what is the La- frangian density for the free E&M held? (dj [Is the Lagrangian a Lorentz scalar field’? How can you easily justify your answer? (e) It is often said that magnetic fields are a consequence of electric fields and relativity. Suppose that in some frame E = 0 every- where but B is not zero. Is there a frame in which E’ not zero and B’ = 0? L. A light wave with Eo perpendicular to the plane of incidence strikes an interface with vacuum as shown in the figure below. The index of refraction in regions 0 and 2 is n, and sin(7) > 4 Find the amplitude, Bs of the wave transmitted to region (2) in terms of Ey. 1. An inductor consists of two concentric conducting cylinders of length d and radii a and 6. Half of the volume between the cylin- ders is filled with a medium of permeability 1, the other half with a medium of permeability j:2 as shown in the figure. Find the fol- lowing for each of the two cases: 1) a current, I, flows parallel to the axis of the cylinder and in opposite directions on the inner and outer conductor, and 2) a current, I, flows counter-clockwise in the inner cylinder and clockwise in the outer. (a) The fields B and H. (b) The total magnetic energy. (c) The inductance L. L. A plane electromagnetic wave with propagation vector kg and po- larization vector €y has electric field E(x,t) = enEpeil*o © =“ and encounters an electron. (a) What is the clectron’s acceleration ¥(t)? (b) What is the time-averaged power (ss) } radiated by the electron Here is an e per unit solid angle into polarization state € . og : 4 dP) dP e . pression for the instantaneous quantity, J5: Gq = 7a lero where ¢* is the complex conjugate of ¢. Assume that the charge is moving slowly. (c) What is the differential cross-section for Thomson Scattering, da __ cutgoingpenwer | {unitsolidangle dey an’ Here ts the definition: 3 = power funttarea (d) How much power is radiated in the direction parallel to eg? 1. A spherical surface of radius A centered at the origin carries a surface charge density given by o(@) = oo cos @ where @ is the polar angle from the z axis. Find the potential at distances r = iR and 7 = 2R from the origin at angles 7/4 and x/2 from the z axis in the z-z plane. Give a symmetry argument for the latter result. La) To determine the self energy, we first need to determine the electric field and, consequently, the potential inside the sphere. We find the electric field inside and ontside to be Qa » Ag R ee R Sr r>k oF Thus, the potential inside is R 3 r 3 Rg E- uf pk arf Feige — 2 ft co Sepr? rn 3e0 3e0 Lr] _ pk Lid erp [Rr eg \3 6 Gen en | 2 6 The sel energy is given by Rp pe [foe Pgh EB 2 cody 2 RTy oy ol 5 Re ir'| dr = [ ; 6 (b) The self-energy of the field is given by u=9 f Bur which is integrated over all space. Thus, we find 2 oR 32 poo U =2ne (2) / rar (2) [ rdr Bea/ Jo Sep Rr 2 a 1. ~2ra (2) : Bef |5 oc 2 wal Qn = rep (< ) R (G+ ) == ke 3ep 5 ' Thus, we see that we get the same result for the self-energy of the sphere and for the self-energy of the field. This is exactly what we expect to be the case as the field is created by our sphere — Rr 0 and, as such, the energy of the field should be the same as the energy of the object that creates it. Furthermore, the equation for the selbenergy of the field is derived from the equation used jor the selenergy of the sphere in part a. AWM) awn} | |. Consider the circuit shown above. (a) Find the impedance to a voltage V of frequency w applied to the terminals. (b) If one varies the frequency but not the amplitude of V, what is the maximum current that can flow? The minimum current? At what frequency will the minimum current be observed? L. A waveguide is made of two perfectly conducting coaxial cylinders with the radiation propagating in the space between them. Take the x direction to be along the axis of the cylinder as shown in the diagram. Assume a solution of the form E(x, y, 2, ¢) = Eoly, z)eflhe ot) Bx, y.z,t) = Baly. zet thet) with Eo = Byéy + E:¢: Bo = Byé, + Bee. where Ey, F., By, and B. are assumed to be real. (a) What are the boundary conditions on E and Bat the conducting suriaces! (b) Show that a mode with both electric and magnetic fields per- pendicular to the axis of the cylinder (transverse electric and magnetic (TEM) modejlie., E,, B, = 0) is consistent with Maxwell's equations and the boundary conditions. (c) What is the velocity of propagation of this mode? (d) Would such a mode be possible without the inner conductor? Explain briefly. 1. A diclectric sphere of radius a and dielectric constant £) is placed in a dielectric liquid of infinite extent and dielectric constant y. A uniform electric field E was originally present in the liquid. Find the resultant electric field inside and outside the sphere. 1. A horizontal magnetic field of strength B (out of the paper) is produced across a square gap. A closed rectangular wire loop with resistance A, negligible inductance, and a mass 7m is allowed to fall with the top of the loop in the magnetic field, as shown. (a) Pind the direction and magnitude of the induced emf as a func- tion of the downward velocity v. (b) Pind the induced current J in the loop asa function of the down- ward velocity. (c) Find the terminal velocity v., of the loop (while the top of the Joop is in the magnetic field) when the gravitational force on the Joop balances the magnetic force. L. A ring of charge @ and radius R is in the 2-y plane and centered around the z-axis. (a) Find the potential along the z-axis. (b) Pind a multipole expansion for the potential everywhere in terms of r (the distance from the otigin) and @ (the angle from the 2- axis). (c) Pind an expansion tor the electric tield E everywhere. (d) A charge q with mass m is confined to move only along the 2- axis. Is these a stable equilibrium point for the charge anywhere on the z-axis? If so, what is the oscillation frequency for small oscillations about the equilibrium position’? 1. A wide slab of dielectric is perpendicular to a uniform external elec- tric field E = Ez. The dielectric contains a very small spherical cavity. What is the electric field at the center of the cavity? 1. A proton is initially traveling at speed wo, a << c, perpendicular to a uniform magnetic field B. It loses energy by radiation. How long does it take to lose half of its initial speed’ 1. A point charge q is situated at the point (x, y, z) = (zo, 0,0), where ap >. (a) Calculate the monopole and dipole moments of this charge dis- tribution. (b) Calculate the electrostatic potential, at an arbitrary point, pro- duced solely by these two moments, the monopole and dipole. (c) What is the exact expression for the electrostatic potential at an arbitrary point? (d) At a distant point on the r— axis, with « >> x9, compare the exact potential with the monopole plus dipole approximation. Which is larger, the exact or the approximate; what, roughly, is the percent difference? l(a} Write the relation between the components of (ct,1r) in two in- ertial frames, with the primed frame moving at relative velocity v in the z direction. (¢ is time, r is position, and ¢ is the speed of light.) (b) Write the relation between the electric field E and the scalar and vector potentials 7 and A for one of the two frames. Repeat for the magnetic field B (c) Find how # can be expressed in terms of variables in the primed frame. Repeat for “. (d) Starting from the fact that both (ct,R) and (U/c, A are four- vectors, find E, and B, in terms of components of E’ and B’ and w, as appropriate. (See part (e) for a check.) (e) For two arbitrary inertial frames, E| =7(E.t+vx Bi) E| =E, Bl = + (Bi —v x Efe’) By = B, A laser in rocket ship P is pointed along the axis, and the laser beam is viewed by instruments in rocket ship @. P and @ are traveling such that their relative velocity is along the y axis, and neither P nor @ is accelerating. Approximate the laser beam by a plane wave. How, explicitly, are the E’ and B’ of the laser beam observed by @ related to the E and B observed by P’? 1. The general expressions for the electric field E and the magnetic field B for a point charge q in arbitrary motion are E = E, + Ea. and B = B,+ Ba with the ‘velocity" and ‘acceleration’ fields being q (1- B°)(R- 8B) dre (1-8 FSRe cB,(r. ) = Bx E, (r,t) _ @ Rx [(F — 8) x al —drecc? (1-8 ABR cB,(r, () = BR» E, (r,t) E,(r, t) = E,(r.() where R = r—r,, r, is the position of g, R= g and all variables on the right hand side of the equations for EB, and E,, as well as Fin all equations are evaluated at the retarded time t — R/e. (a) A positron of energy 5 GeV is in a circular orbit of radius p, and the point of observation r = ry is in the orbit plane at a distance 2p from the center of that circle. At the time of observation t= ty, the field &, at rp is a maximum. i. What is the ratio E/E, then? An approximate numerical result is expected. ii. Make a diagram showing the position of the positron at time ty and the directions of all four fields at rg and fp. (b) Show how some part of the equations for E., E,, B,, and By reduces to Coulomb's law. 1. An object has a fixed polarization density P(x) and no free charge. The medium outside the object is a vacuum. (a) Write down the equations which determine the electric field B. These equations should include differential equations, boundary conditions, and asymptotic conditions. (b) Repeat part (a) for the D field. (c) What is the E field everywhere for an infinite cylinder uniformly polarized parallel to its axis? (d) Repeat (ec) for the D field. (e) Sketch the E field for a semi-infinite cylinder uniformly polarized parallel to its axis. (f} Repeat (e) for the D field. (g) Consider the D field on the azis of a semi-infinite cylinder uni- formly polarized parallel to its axis. How does the field at the end compare with the field deep inside the cylinder and far from the end? A quantitative comparison is required. 1. In parts (a) - (c) the system consists of a particle of mass m and charge € moving in an external field B(r) = V x A(r), where both B and A are time-independent. (a) Given that hk = Bart where Pyne = P—=A, and p is the usual momentum operator, —ihV in the position representation, show that the velocity operator v= me aps di 18 V, (b) What is the appropriate quantum mechanical operator version of the classical foree <(v x B)? State the critical property of the quantum mechanical operator version which determines its form. J op = jn Pimech- (c) Using the quantum mechanical equation of motion, verify that Prncch _ [Fw x B)] t Cc (d) This part is classical, not quantum mechanical. Show that if A is independent of x as well as #, A = Aly, 2), and if rect dp" dé e (v x B), then the 2 component of the canonical momentum, mech, e Pr =p +A, c is conserved, i.e., aps =0. 1. The distance between centers of two conducting spheres is /. Each sphere is of radius R. (a) In the limit in which | >> A determine the capacitance of the two sphere system. (b) Consider next a first order correction to (a) above. In this ap- proximation, each sphere appears as a point charge to the other. 1. A uniform, static electric ficld with magnitude E points in the z-direction. At ¢ = 0 a particle with mass m and charge ¢ has a velocity w in the y-direction. Find v,(t) and v,(t). (Include relativistic effects.) 1. A particle of charge q moving on a trajectory r’(#’) produces, at position r and time ¢, an electric field Eir,é) = B,(v,¢) + E,(r,d) where the “velocity” and “acceleration” fields are given by R-p E,(r.t)=4 Fae PIOR Hat-Rfe and Rx (R-B x B E,(r,t)=¢ [See] Hot—Rfe with “adr, dB op yy B= ap Paap? =0-a7y', R=r-r,and K=1-R#- g. The magnetic field at (r, #) is given in terms of E by B = R x E. (a) (20 pts.) Show that the velocity field makes no contribution to the energy radiated away by the moving charge. (b) (20 pts.) For a negatively charged particle moving at constant velocity, show that E, points toward the present (not retarded) position of the charge. For the rest of the problem, assume that $ << 1 and ignore E,. (c) (20 pts.) Find the Poynting vector S(r,¢) in terms of a = ¢8 (d) (25 pts.) Find the time-averaged rate of energy radiation for a particle of charge g moving in a circle of radius rp with constant angular velocity wo. (e) (15 pts.) Consider a charge distribution composed of N charges, each of magnitude q/N, equally spaced along a circle of radius ry and all moving around the circle with angular velocity wp. ‘What would be the power radiated by this system in the limit N — oc? Explain your result. 1. The upper half of the figure shows electric field lines and equi- potential surfaces for two infinitely long cylindrical conducting thin shells with vacuum between them, at a difference of potential of 100 volts. Each segment in the upper half is a curvilinear square, defined as a planar geometric figure formed by four curved lines that intersect at right angles, such that successive subdivisions will yield squares in the limit of a large number of subdivisions. The lower half of the figure is an incorrect version of the same thing. All of the questions, except the first, concern the upper half of the figure. Explain each of your answers using only the figure and fundamental ideas (do not solve the problem analytically). The permittivity of free space is 8.8 picofarad,'m. (a) What is the most obvious physical clue that the lower half of the figure is incorrect’? (b) Estimate the electric field (in volts/em) at the pont P1. (c) Estimate the potential at P). (d) Estimate the charge density (in C/em?) on the surface of the inner cylinder at Py and P3. (e) Estimate the capacitance (in pF/em) per centimeter length of the two cylinders. (f} How would the figure change if the vacuum were replaced by a dielectric with the dielectric constant of 2, still having 100 volts between the conductors? (g) For the dielectric, how would each of your answers to (b) through (e) change? 1. A cylindrical, parallel-plate capacitor is filled with a homogeneous, linear dielectric with permittivity ¢ and permeability jo (SI units). The separation of the plates is Z and the radius of the capacitor is R. There is a uniform field, D(¢) inside the capacitor which increases linearly with time due to a constant current flowing onto the ca- pacitor: Dit) =ta (where a is a constant vector) In the following neglect the fringe field outside the dielectric and leakage currents inside the dielectric. (a) Find the H field as a function of r, the radial distance from the central axis inside the dielectric. (b) Find the Poynting vector, 8, as a function of r inside the dielec- tric. (c) Use the Poynting vector to find the energy flux into a cylinder of the dielectric of length Z and radius r on the central axis (see figure). (d) Starting from the expression for the field energy density, caleu- late the rate of increase of teh field energy inside the cylinder of Part (c), and verify that energy is conserved. . An important problem in atomic, nuclear, and particle physics is the calculation of the energy lost by ions as they traverse matter: eg., protons through multiwire proportional chambers or uranium nuclei through ionization chambers. Assume the ion of charge ze and rest mass Af is traveling with velocity v = vt. Consider the ‘collision’ (electromagnetic interaction) between Jf and an elec- tron, rest mass m and charge —e. Assume further that v is much larger than the characteristic velocity of the electron in its atomic orbit, and neglect the electronic binding energy; i.e, assume the electron is free and at rest, and remains so during the collision time. The ion’s distance of closest approach is 6. (a) Derive an expression for the transverse momentum impulse im- parted to the electron: (i.e., for dp|z). HINT: A relativistic or non-relativistic (electrostatic) treatment yields the same answer. (b) What is the energy lost by the ion in the collision’? (c) If there are n atoms per unit volume, each with Z electrons, and 6 ranges from bypin tO bya, derive an expression for —3—, the energy loss of the ion per unit distance of matter traversed. 1. General Octagon has developed a ‘Rail-Gun’ to lob a projecticle of mass MM a great distance at his arch enemy, Commander Gremlin. The Rail-Gun consists of two parallel cylindrical rails of diameter a with axes separated by a distance d. A current J passes up on rail, passes through the shell and returns down the other rail. The rails are frictionless and are infinitely long behind the shell for our purposes. The current through the shell passes in a line across the rear. See the drawing. (a) (35 pts.) Calculate the net force on the shell. (b) (5 pts.) If Octagon has a maximum current J) available to launch the shell to terminal velocity v, how long must the rails be in front of the starting position? 1. When light hits the smooth surface of a thin glass plate at nor- mal incidence, about 4% of the intensity gets reflected back, and also the phase of the wave function gets changed by 180°. After penetrating the depth of the glass, again 4% of the light gets re- flected from the bottom surface of the glass, but with no sudden phase change. What percent of the incident light intensity is re- flected back (i.e., combining the reflection from the top surface and the bottom surface; here we ignore multiple reflections inside the glass); (a) when the thickness is glass? (b) when the thickness is glass? (c) when the thickness is a full wavelength of the light in the glass? ; of the wavelength of the light in the of the wavelength of the light in the 1. A plane interface separates two linear dielectric-permeable media with D, E, B, and H fields in Medium 1 and D’, EB’, B’, and H’ fields in Medium 2. cs and K, are the surface charge and current densities in the plane of the interface. 7 is a unit vector normal to the interface as shown. (a) Derive the expressions relating the normal components of the D fields, the normal components of the B fields, the tangential components of the E fields, and the tangential components of the H fields across the interface. (b) Show that when Medium 2 becomes a perfect conductor and when there are no static fields present, then your results of part (a) reduce to 4 A D=4drosaxE=0, @ B=0,andaxH=—Ks ce (c) A plane-polarized plane wave E,(r.t) = Eye 7" is incident obliquely onto the plane face of a perfect conductor where Ep, is real, constant, and perpendicular (out of the paper) to the plane of incidence as shown. Complete the diagram and show how you obtain the directions of the vectors Hy;, Ey,, Hy,, and k, where 7 refers to the incident wave and r refers to the reflected wave. (d) Find the magnitudes of Ho, En-, Ho,-, and es in terms of Eq: and ¢ and yu of the incident medium. Also find Ks (including its direction). 1. We assume here that a metallic conductor can be simulated by a uniform gas of electrons of mass 7 and number density n moving in a positive ion background having the same number density. For a given electron, the average time between collisions with an ion is 7, and we assume that the electron comes to rest after a collision. We ignore electron-electron collisions. We further assume that there is no static electric or magnetic polarization. When subject to an electric field Bir, t) = e(r, te, the a.c. electric conductivity can be approximated by ay ne ow) =—2 where a= l-iwr (a) Briefly, what is the meaning of the expression I(r, 8) = o(w)E(r,t)? For the two limiting cases wr << 1 and wr >> 1, discuss the behavior of J(r, ¢) relative to E(r, 4), and for each case give semi- quantitative physical arguments explaining why such behavior is expected to occur. (b) Show that in the metal (1.2) satisfies 2 Ve(r,w) +2 Sele.) =0, where metal, = 14 84) is the ac. dielectric constant of the (c) Under certain conditions the metal may be transparent, with waves propagating, or reflecting, with almost complete reflection of an incident wave. For the case wr >> 1, find the frequencies w, relative to the plasma frequency wp = where the metal will be transparent and the frequencies where it will be reflecting. (d) Inside the metal the electric field can be written as Er, £) = Ey(tew)e"® | +t), where Ep, k’ and a are all real. Find the energy lost to heat per cycle per unit volume in terms of Ep, w, and constants. 1. An infinite plane interface separates space into two regions. Re- gions 1 and 2 are filled with linear dielectric materials with permit- tivities of €) and 2 respectively. A point charge q is a distance A in front of the interface in region 1 as shown. (a) Find the total induced bound charge o;, on the interface. Find the image charge(s) which would enable one to find the clectric field in region 1, and find the image charge{s) which would enable one to find the electric field in region 2. (b) In order to use image charges in part (a), one needed to invoke the uniqueness theorem which says that the electric field, E(r), in a given region of a linear dielectric is unique provided the normal components of E on the boundary of the region are specified. Prove this uniqueness theorem. 1. Consider a cylindrically symmetric magnet with circular pole pieces of radius R and gap d. The static B field is predominantly along the axis of symmetry, or z direction of a cylindrical polar coordinate system centered in the median plane of the magnet. Further, consider a particle of charge g and mass m, with kinetic energy mv? /2 with uw < &. (a) If.B = B) is parallel to the axis and constant in the gap, find the frequency, wy, of circular orbits in the median plane. Find the number of revolutions this particle will undergo before reaching the pole tip if it makes a small angle a, with respect to the median plane. (b) Consider orbits well away from the axis, but still in the gap where the z- component of the B field, B, is a decreasing function of r, the distance from the axis. Specifically, let B.(r) = B/r* in this region, B and ¢ are constants. Show that particles off the me- dian plane experience a force directed toward the median plane and calculate the magnitude of the force. Find the frequency of oscillations about the median plane in terms of wp. 1. Calculate the monopole, dipole and quadrupole components of the electric potential at a point on the z axis for a point charge +e situated at r= (a, y, z). 1. Two straight parallel beams of protons are a distance d apart. Each beam has negligible radius, the same charge per length 4, and the same current J, since every proton has the same constant velocity v. Calculate the force per length on one beam due to the presence of the other beam. Does that force per length represent. an attraction between the beams? Repulsion? What happens as voc? l(a) Using V x [v x Sere) and neglecting the surface inte- gral show that the current source can be written as 1 3 Txt 1 Vi Jix'.t) Jix.t) = Vx vx fay A 4 - feet |x — x’ an jx —x’| dn (b) Interpret the two terms on the right. Hint: Vx (Vx F)-V(V F)-WF 1. Given that the vector potential of the carth's magnetic field is A; = Ag = and Ay = ae where js is the magnetic dipole moment, show that a charged particle of mass m and charge e, initially at ro, with velocity v (without a @ component), would be trapped in a region in space. sin@ 1 > 1 rr rrosin@| — az sin an aE where ap = 4/ =. 1. A plane monochromatic electromagnetic wave in vacuo falls nor- mally onto a plane semi-infinite dielectric slab. The permittivity and permeability of the dielectric are ¢ = ¢-ep and pp = poppin re- spectively. (a) For each of the following seven quantities find the ratio of their value in the dielectric to their value in vacuo. i. velocity, v ii. angular frequency, w iii. wavelength, A iv. propagation constant, k v. total electric field strength (£) near the interface vi. index of refraction, n vii. ratio of the total electric field to the total magnetic field near the interface (FE/H). (b) Find the reflaction and transmission coefficients E, r= |=] and i Ey |B (c) What constraint does conservation of energy impose of r and ¢? 1. A homogeneous spherical shell of mass Mf is shown below. Find the gravitational field (that is, force per unit mass) Fe(r), in the three regions I, II, III shown. m /\ 1. A particular physical system consists of a sphere of radius R which contains charged particles. The potential outside the sphere is found to be 2 P(r, 6.0) = S¥;'(00) where @ is an experimentally determined constant and Yj’ is the spherical harmonic 1 fs ¥3'(6, a) = a = (3 cos’ @ — 1) The charge density, p, is zero outside the sphere and is proportional to r times a function of @ inside. Find p = p(r@). 1. A particle of mass m with electric charge e moves in an clectro- magnetic field. Let its position be r and velocity be v = a The electromagnetic field is derived from the potentials A and @ in the usual way. (a) Show that the following lagrangian reproduces the appropriate equation of motion (the Lorentz force on the particle). 1 : L==me+"v A-ed 2 c (b) From this lagrangian develop the corresponding hamiltonian. (Be sure to distingush the canonically conjugate momentum to the position coordinate from the mechanical momentum.) (c) Consider r = (q),q2,q3), and a non-singular transformation Q: = Oxla.t). Express the new canonically conjugate momenta FP, in terms of the old conjugate momenta and the transofmra- tion. l(a} Write Maxwell’s equations in terms of the field-strength tensor Fe", Assume the medium is free space (D = E, B = H) with sources. (b) Derive a consistency condition for the four-vector current. (c) What is the relation between F?” and the four vector potential AH? (d) Show that the four-vector potential is not unique. Give the most general term which may be added to the potential which leaves the field strength tensor unchanged. How is this lack of uniqueness usually handled? (e) Derive the equivalent of Maxwell's equations in terms of A“. 1. A conducting sphere has radius A and carries a total charge Q. Half of the sphere is immersed in a semi-infinite diclectric with dielectric constant © and half lies in a vacuum. What is the net force pulling the sphere into the diclectric? 1. Accelerated charges radiate. For a point charge moving nonrela- tivistically, the radiation of the electric field is e fax(nxa) E= 5 |—,—_ afr with: 7 is the unit vector from the charge to the observation point. ais the acceleration of the particle. Ris the distance from the change to the observation point. e is the charge of the particle. Assume it is positive. {| means evaluate at the retarded time. (a) The observation point is on the positive x-axis. At the retarded time, the particle is at the origin with an acceleration in the positive y direction. What is the direction of the electrical field at the observation point? (b) Evidently, a point charge moving in a circle radiates. On the other hand, when a uniform circle of charge is rotated about: it’s symmetry axis, it does not radiate. But the uniform circle of charge can be arbitrarily well approximated by an assembly of infinitesimal point charges infinitesimally eparted on the circle. Even though each of these by itself would radiate, the circle does not. Explain. Use the form of E above to show how your argument works. It is sufficient to restrict your attention to the plane of the circle and to take A very much greater than the radius of the circle so that all the points on the circle have approximately the same distance from the observation point, direction to the observation point, and retarded time. 1. A beam of light with intensity Jp and frequency 1 is directed along the positive z-acis and reflects off a perfect mirror which is moving along the positive z-axis with a velocity vp. The beam impinges normally on the receding mirror. What is the intensity J and frequency y of the reflected light in terms of Jo and un?

You might also like